PT50.S2.Q14 - Government funds researches....

jovytai417jovytai417 Alum Member
edited February 2018 in Logical Reasoning 10 karma

Here is my analysis of the answer choices. Grateful for commenting on my thoughts, please! Thank you!

(A): Both disagreed. Cynthia - the reason to be funded by Gov is to further theoretical knowledge not unforeseen practical applications. Luis - the "expected" to yield practical applications in the stimulus is more definite that "may have unforeseen.." in AC.

(B): Luis disagrees; Cynthia - not known because we don't know what project does she think the Gov should not fund. In the stimulus, we only know Cynthia would agree that the government should fund researches that further the theoretical knowledge, but that does not mean the Gov should not fund projects that have practical application.

(C): Luis - not known because the only thing we know from stimulus is "Gov fund projects --> research that is expected to yield practical applications" (ie. every gov funded research should have practical application), but that does not mean that every research that has practical application should be funded by Gov. As for Cynthia, I am confused because I am not sure if "research project in theoretical science" in AC is equivalent to "research project seeks to further theoretical knowledge of nature" in the stimulus. But in either way, the AC should be eliminated.

(D) Not known because we don't know if the new technologies will help further theoretical knowledge of nature or yield practical applications

(E) Cynthia agrees and Luis disagrees.

Are my interpretations correct?

Thanks!

https://7sage.com/lsat_explanations/lsat-50-section-2-question-14/

Admin edit: title and link

Sign In or Register to comment.